Investments Practice Exam 2
Joe is a 24% federal income taxpayer. His capital-gains rate and qualified dividend rate is 15%. Joe bought 100 shares of ING for $12 per share 10 months ago. ING paid a dividend of $80 during Joe's holding period. Joe just sold all the stock for $20 per share. What are Joe's tax consequences for the year? A) $80 taxed at the LTCG rate, $800 taxed at the ordinary income rate B) $80 taxed at the ordinary income rate, $800 taxed at the LTCG rate C) $720 taxed at the ordinary income rate D) $880 taxed at the LTCG rate
A) $80 taxed at the LTCG rate, $800 taxed at the ordinary income rate The dividend is a qualified dividend taxed at the LTCG rate. The gain is a short-term capital gain and is taxed at ordinary income tax rates. SP $2,000 PP ($1,200) Gain $800 (STCG)
A fixed-income fund has a one-year holding period return of 6% while the risk-free rate is 2%. Measures of risk for Hawk include a beta of 0.6 and a standard deviation of 10%. The estimate for the Sharpe ratio is closest to: A) 0.4. B) 0.8. C) 6.7. D) 10.0.
A) 0.4. Sharpe = Actual Return - Risk-free Rate / Standard Deviation. Sharpe = (6 - 2) / 10 = 0.4
A mutual fund's returns over the past 4 years were -4%, -15%, 13%, and 13%. What was its geometric mean return over these years? A) 1.03% B) 1.75% C) 2.08% D) 11.17%
A) 1.03% GMR = [(1 + -4%) × (1 + -15%) × (1 + 13%) × (1 + 13%)](1/4) − 1 GMR = [(0.96) × (0.85) × (1.13) × (1.13)](1/4) − 1 GMR = [1.0420](1/4) − 1 GMR = 1.0103 − 1 = 0.0103 = 1.03%
Four years ago, an investor contributed $5,000 to an investment portfolio. A year later, the investor contributed another $1,000. Another year later, the investor withdrew $1,000. Last year, the investor contributed $2,000. Today, the portfolio is worth $10,000. What was this investor's dollar-weighted return over the past 4 years? A) 11.18% B) 37.50% C) 100.00% D) 120.00%
A) 11.18% 5,000 [+/-] [CFj] 1,000 [+/-] [CFj] 1,000 [CFj] 2,000 [+/-] [CFj] 10,000 [CFj] [SHIFT] [IRR/YR] The correct answer is about 11.18%.
According to the Capital Asset Pricing Model (CAPM), if the risk-free rate is 3% and the market portfolios expected return is 10%, what is the expected return of a stock with a beta of 1.4? A) 12.8% B) 12.9% C) 13.0% D) 14.0%
A) 12.8% Expected return = rf + β (rm - rf) Expected return = 3% + 1.4 (10% − 3%) = 3% + 1.4 (7%) = 3% + 9.8% = 12.8%
Brandy is considering purchasing an 8-year bond that is selling for $700. What is the current yield for this bond if it has a 6% coupon, paid semiannually? A) 8.57% B) 9.32% C) 10.17% D) 11.92%
A) 8.57% Current yield = $60/$700 = 8.57%
An investor is close to retirement and is worried about the possibility of his stock portfolio suddenly losing value. He has heard that certain options strategies can limit his downside risk, but he is hesitant to pay premiums each month. Which options strategy, if structured appropriately, could limit his downside risk without costing him anything out of pocket? A) A collar B) A protective put C) A covered call D) A long straddle
A) A collar A collar combines a protective put and a covered call. A zero-cost collar is created when the premiums received from the short call offset the premiums paid for the long put.
Which of the following is an example of an alternative investment? A) A limited partnership in a private equity firm B) A callable corporate bond C) A leveraged mutual fund D) A STRIP
A) A limited partnership in a private equity firm Private equity is an alternative investment.
According to the Black-Scholes option pricing model, the value of a call option will decrease under which of the following conditions, all other things equal? A) As the exercise price increases B) As time to maturity increases C) As the underlying stock's price increases D) As interest rates increase
A) As the exercise price increases The higher the exercise price, the less likely the option will ever be in the money. Therefore, higher exercise prices will lead to decreased call option prices.
Which of the following is (are) included in a mutual fund's expense ratio? 1) 12b-1 fees 2) Front-end loads A) I only B) II only C) Both I and II D) Neither I nor II
A) I only While 12b-1 fees are included in a fund's expense ratio, front-end loads are not.
Over the past 4 years, a mutual fund had returns of 5%, -10%, -15%, and 10%. What was the arithmetic mean and standard deviation of this fund's returns? A) Mean = -2.50%, SD = 11.90% B) Mean = -8.16%, SD = -4.08% C) Mean = -8.16%, SD = 4.08% D) Mean = -2.50%, SD = -11.90%
A) Mean = -2.50%, SD = 11.90% The mean is -2.50%, and the standard deviation is close to 11.90%. You can calculate these using the ∑+ key on a financial calculator
Which of the following statements about common stock is correct? A) Most common stockholders have a right to vote on corporate matters. B) The number of votes per share will be higher for Class A shares than for Class B shares. C) Bondholders generally have more votes per bond than common stockholders have votes per share. D) If a dividend payment is missed, a company must repay the missed dividend to common stockholders.
A) Most common stockholders have a right to vote on corporate matters. Most common stockholders have a right to vote on corporate matters. Statement (B) is incorrect because, while different classes of stock generally offer different voting rights, there is no standard convention. Statement (C) is incorrect because bondholders generally do not have any voting rights. Statement (D) is incorrect because this rule applies to cumulative preferred stock.
Which securities law regulates the offering and sale of securities in the primary market and ensures more transparency in financial statements? A) The Securities Act of 1933 B) The Securities Exchange Act of 1934 C) The Investment Advisers Act of 1940 D) The Investment Company Act of 1940
A) The Securities Act of 1933 The Securities Act of 1933 requires disclosures of new securities in the primary market. The Securities Exchange Act of 1934 focuses on the trading of securities in the secondary market. The Investment Advisers Act of 1940 regulates investment advisers and requires registration with the SEC for firms or any individual advisers with assets under management exceeding $100 million. The Investment Company Act of 1940 forms the backbone of financial regulation and established the foundation for mutual funds and hedge funds.
If interest rates increase, then bond prices will _______ while the returns on reinvested coupons will _______. A) decrease, increase B) decrease, decrease C) increase, decrease D) increase, increase
A) decrease, increase There is an inverse relationship between interest rate risk and reinvestment rate risk. If interest rates increase, bond prices will fall, while reinvest coupons will experience increased returns.
An investment manager routinely uses technical analysis to outperform his benchmark portfolio. He also finds that fundamental analysis and a little bit of insider trading (which is illegal) provide improved returns. Based on this information, he is investing in a market that is: A) inefficient. B) weak-form efficient. C) semi-strong form efficient. D) strong-form efficient.
A) inefficient. Technical analysis should not provide improved returns in any type of efficient market. This market is inefficient.
A bond has a 5% coupon with semiannual payments and matures in 10 years. If the yield to maturity (YTM) for this bond is 4%, what is the value of the bond? A) $1,075 B) $1,082 C) $1,090 D) $1,097
B) $1,082 P/YR 2 FV $1,000 N 2 × 10 = 20 i 4% PMT 5% × 1,000/2 = $25 PV $1,082
Jenny purchases shares of Shrimp Co. for $32.50 using a margin account with a 50% initial margin, assuming the maintenance margin is 35%. At what price will Jenny receive a margin call? A) $16.25 B) $25.00 C) $27.50 D) $31.50
B) $25.00 Debt/(1 − Maintenance Margin) = Account Value. ($32.50 − $16.25)/(1 − 0.35) = $25
Billie originally purchased display cases for her business at $15,000. They were fully depreciated by the time Billie retired. Billie died last month, and the display cases were valued in her estate at $8,000. If Billie's daughter, Kelly, inherits the display cases and sells them for $8,500 2 months after Billie's death, what is the income tax treatment on the sale? A) $500 ordinary income B) $500 long-term capital gain C) $500 short-term capital gain D) $8,500 long-term capital gain
B) $500 long-term capital gain Even though the display cases were Section 1231 assets in Billie's hands and were subject to depreciation recapture, once they ran through Billie's estate, they qualified for a Section 1014 step-to fair market value in basis, which eliminates the recapture potential. If Kelly sells the cases for $8,500 2 months after Billie's death, her gain is $500 due to the step-up in basis. Since all assets passing through an estate and receiving a step-up in basis qualify for long-term capital-gains treatment, the gain will be characterized as a long-term gain.
Jack is considering purchasing a 6-year bond that is selling for $1,150. The bond can be called in 3 years at $1,040. What is the YTC for this bond if it has a 9% coupon, paid semiannually? A) 4.63% B) 4.82% C) 5.03% D) 5.99%
B) 4.82% P/YR = 2 PV = -$1,150.00 N = 6 FV = $1,040.00 PMT = $45 I/YR = 4.82
The creation of which of the following is most often linked to the increased popularity of investment companies? A) Roth IRAs B) 401(k) Plans C) The Investment Company Act of 1940 D) The Investment Advisers Act of 1940
B) 401(k) Plans The growth of 401(k) plans in the 1970s and 1980s required individuals to save for their retirement. Investment companies provided an opportunity for investors to save for retirement without dedicating much time and expense to managing their investments.
Peter has a fixed-income portfolio that consists of Bond A, Bond B, and Bond C. The bonds have durations of 4, 6, and 10, respectively. If Peter has 50% invested in Bond A and 25% invested in each of the other two bonds, what is the duration for the portfolio? A) 5.5 B) 6.0 C) 6.7 D) 7.2
B) 6.0 (0.5 × 4) + (0.25 × 6) + (0.25 × 10) = 2 + 1.5 + 2.5 = 6
A portfolio includes $4,000 in TOP stock, $3,000 in MID stock, and $3,000 in BOT stock. TOP, MID, and BOT have expected returns of 9%, 7%, and 5%, respectively. What is the weighted expected return for this portfolio? A) 7.0% B) 7.2% C) 7.4% D) 7.6%
B) 7.2% Divide each return by total return ($10,000) to get the weights. Then solve like traditional weighted avg return. Ex: ($4,000/$10,000) = .4 --> (.4 times 9) = 3.6 (.3 times 7) = 2.1 (.3 times 5)= 1.5 3.6+2.1+1.5= 7.2
This year, an investment returned a 10% rate of return, while the CPI measured 2%. What was the real rate of return on this investment? A) 5.00% B) 7.84% C) 8.00% D) 12.20%
B) 7.84% [(1.10/1.02) − 1] × 100 = [1.0784 − 1] × 100 = 0.0784 × 100 = 7.84%
Beverly engaged in several capital transactions this year. She had a short-term capital gain of $400, a short-term capital loss of $600, a long-term capital gain of $800, and a long-term capital loss of $500. How will Beverly report these items on her income tax return? A) Beverly will report a net short-term capital gain of $100. B) Beverly will report a net long-term capital gain of $100. C) Beverly will report a net short-term capital loss of $200 and a net long-term capital gain of $300. D) Beverly will report ordinary income of $100.
B) Beverly will report a net long-term capital gain of $100. Short-term capital losses can be netted against short-term capital gains, and long-term capital losses can be netted against long-term capital gains. Since there is a short-term loss of $200 and a long-term gain of $300, the short- and long-term summary results can be netted together. In this case, since the long-term capital gain was larger on an absolute basis, the character of the resulting net gain is a long-term capital gain.
Ignoring trading commissions and taxes, which of the following outcomes is possible in a single options trade? A) Both the buyer and seller can profit. B) If the buyer profits, the seller loses money. C) Both the buyer and seller can lose money. D) The answer depends on whether the option is a put or a call.
B) If the buyer profits, the seller loses money. Options are a zero-sum game. If the buyer profits, the seller loses money (and vice versa).
When making an asset selection decision, an analyst elects to use the relative valuation approach. Which of the following is a disadvantage of this approach? A) It requires a large number of assumptions to produce an analysis. B) It is difficult to identify acceptable peer groups for comparison. C) It rarely estimates prices that are close to actual market prices. D) It produces analyses that are confusing and counter-intuitive.
B) It is difficult to identify acceptable peer groups for comparison. With the relative valuation approach, it is often difficult to identify acceptable peer groups for a given company. Analysts must balance firm size, capital structure, and location.
A real estate investment trust (REIT) manages a portfolio of office buildings that lease space on their ground floor to small shopping centers. In what type of property does this REIT invest? A) Retail B) Mixed-use C) Commercial D) Industrial
B) Mixed-use Mixed-use property is the name given to buildings that house both office space (commercial) and retail property.
Samantha and Jordan decide to open a sports equipment store together because of their love of the outdoors. They each own 50% in SportsCrazy, LLC, which is taxed as a partnership. Jordan manages the business. Samantha has a thriving tax practice and therefore does not participate in the operation of the business. Which of the following is true? A) Only the income distributed to Jordan is considered passive income. B) Only the income distributed to Samantha is considered passive income. C) The income distributed to both Jordan and Samantha is considered passive income. D) The income distributed to both Jordan and Samantha is considered active income.
B) Only the income distributed to Samantha is considered passive income. The distributive share of the partnership income that Jordan receives is considered active income since Jordan materially participates in the conduct of the SportsCrazy business. Samantha's distributive share of the partnership income is treated as passive income/loss since Samantha is not materially participating in the conduct of the business activities.
On September 20 of Year 1, Rene purchased 1,000 shares of Carter, Inc., common stock for $40,000. He sold the shares for $50,000 on September 20 of Year 2. Which of the following statements correctly identifies the tax consequences of this transaction? A) Rene will recognize a $10,000 ordinary gain on the sale. B) Rene will recognize a $10,000 short-term capital gain on the sale. C) Rene will recognize a $10,000 long-term capital gain on the sale. D) Rene will not be required to recognize the gain on the transaction.
B) Rene will recognize a $10,000 short-term capital gain on the sale. Rene's holding period equals one year. Since it is not more than one year, it does not qualify for long-term capital gain treatment.
Which of the following statements about the prices in a futures contract is correct? A) As the contract nears its expiration date, the spot price and futures price diverge. B) The futures price is the price of the asset at some time in the future. C) Once a futures contract is purchased, its spot price is fixed. D)A spot price is to a forward contract as a futures price is to a futures contract.
B) The futures price is the price of the asset at some time in the future. In a futures contract, the futures price is the price of the asset at some time in the future. Statement (A) is incorrect because, as the futures contract nears its expiration date, the spot price and futures price converge Statement (C) is incorrect because a contract's futures price is fixed once it is purchased. Statement (D) is incorrect because a spot price and futures price apply to both forward and futures contracts.
Ace, Inc., has net earnings of $3.6 billion this year. It has 600 million shares of common stock outstanding, and it paid 75 cents per share per quarter in dividends this year. Which of the following is correct? A) The retention ratio equals 33.33%. B) The payout ratio equals 50.00%. C) The payout ratio equals 33.33%. D) The payout ratio equals 66.67%.
B) The payout ratio equals 50.00%. The dividend per share equals $3.00. The EPS equals $6.00, which is found by dividing net earnings by outstanding shares. The payout ratio = dividend/EPS. $3.00/$6.00 = 50%
Based on a normal distribution, 95% of all outcomes for investment returns should fall within: A) one standard deviation from the mean. B) two standard deviations from the mean. C) three standard deviations from the mean. D) four standard deviations from the mean.
B) two standard deviations from the mean.
A large company expects to generate $800 million in operating cash flows during the next year. It estimates its long-term dividend growth rate to be 3% and it has 80 million shares outstanding. What is the intrinsic value of this company if the required rate of return is 7%? A) $100 B) $143 C) $250 D) $333
C) $250 Intrinsic Value = Total Market Value / Outstanding Shares Total Market Value = Operating Cash Flow / (Required Rate of Return - Dividend Growth Rate) Total Market Value = $800,000,000 / (0.07 − 0.03) = $800,000,000 / 0.04 = $20,000,000,000 Intrinsic value = $20,000,000,000 / 80,000,000 = $250
You buy a call option on a stock for $3 per share. The strike price is $75, and the current price of the stock is $65. If on the expiration date the stock's price is $74, your net profit per share (excluding commission and taxes) is: A) -$10. B) -$7. C) -$3. D) $2.
C) -$3. This option expired out of the money, so you will lose the $3 premium.
An investor receives a premium of $5 for writing a put option with a strike price of $105 on a stock currently trading for $115. At the option's expiration, the stock trades for $95. What is this investor's profit or loss per share? A) -$15 B) -$10 C) -$5 D) $15
C) -$5 The option will be exercised, requiring the investor to purchase the stock for $105 even though it is only worth $95. This $10 loss is offset by the $5 premium received, leading to a loss of $5 per share.
TBT stock pays a $3.00 dividend semiannually and has earnings per share of $9.00. If the stock is currently trading at $60, what is the dividend yield percentage? A) 5.00% B) 33.33% C) 10.00% D) 66.67%
C) 10.00% The dividend yield percentage is equal to the total annual dividends per share divided by the stock price. ($3 × 2)/$60 = 10%
If an investment pays a fixed 7% rate of return each quarter, what is its annualized return? A) 7% B) 28% C) 31% D) 49%
C) 31% Annualized return = (1 + 0.07)4 − 1 = 1.074 − 1 = 1.3108 − 1 = 0.3108, or about 31%
The Arctic Fund earns 10% during the year while the risk-free rate is 2%. The Pacific Fund has a beta of 2 and a standard deviation of 8%. The Treynor Ratio is closest to: A) 1.0 B) 1.3 C) 4.0 D) 5.0
C) 4.0 Treynor = (Portfolio Return − Risk-free Rate) / Beta Treynor = (10% − 2%) / 2 = 4
A coupon bond that pays interest of $40 annually has a par value of $1,000, matures in 12 years, and is selling today at $980. What is the yield to maturity for this bond? A) 3.72% B) 3.87% C) 4.22% D) 4.39%
C) 4.22% N 12 PMT $40.00 PV ($980) FV $1,000.00 I 4.22%
A portfolio consists of two uncorrelated assets: A and B. Using the information in the table below, calculate this portfolio's standard deviation: Asset Weight Mean Return Standard Deviation Portfolio A: 80% Weight, 6%Mean Return, 8%SD Portfolio B: 20% Weight, 4% Mean Return, 6% SD A) 1.63% B) 2.76% C) 6.51% D) 7.60%
C) 6.51% Portfolio SD = [(0.802 × 8%2) + (0.202 × 6%2)]1/2 Portfolio SD = [(0.64 × 64%) + (0.04 × 36%)] 1/2 Portfolio SD = (40.96% + 1.44%)1/2 Portfolio SD = (42.40%)1/2 = 6.51%
A taxpayer in Georgia is in the 24% federal tax bracket and 4% state income tax bracket. Which of the following bonds has the highest after-tax yield for this taxpayer? A) A Treasury bond paying 4.1% B) A corporate bond paying 4.3% C) A Georgia municipal bond paying 3.3% D) A New York municipal bond paying 3.4%
C) A Georgia municipal bond paying 3.3% Municipal bonds are not subject to federal income tax but are subject to state income tax if they are not issued by the taxpayer's state of residence. Treasury bonds are subject to federal income tax only. Corporate bonds are subject to federal and state income tax. Treasury: 4.1% × (1 − 0.24) = 3.116% Corporate: 4.3% × (1 − 0.28) = 3.096% GA Bond: 3.3% × (1 − 0.00) = 3.3% NY Bond: 3.4% × (1 − 0.04) = 3.264%
When an investor sets a bond portfolio's duration to be equal to the investor's time horizon, they reduce the portfolio exposure to: 1) interest rate risk. 2) reinvestment rate risk. A) I only B) II only C) Both I and II D) Neither I nor II
C) Both I and II A bond is immunized if its duration is matched to the timing of the investor's cash flow needs. An immunized portfolio has less exposure to both interest rate and reinvestment rate risks.
The decision of whether to go public is an important one for private companies. Which of the following statements is (are) a disadvantage of going public? 1) Loss of some control over business operations 2) The obligation to file annual and quarterly reports to the SEC A) I only B) II only C) Both I and II D) Neither I nor II
C) Both I and II Companies that go public have less control over their business operations and must file reports to the SEC.
An investor seeking relatively low default risk investments combined with some type of favorable tax treatment and inflation protection should consider which of the following? A) Revenue municipal bonds B) Series EE savings bonds C) Series I savings bonds D) Series HH savings bonds
C) Series I savings bonds Series I savings bonds offer inflation protection and are default-risk-free. Muni bonds and Series EE savings bonds generally do not offer inflation protection. Series HH bonds are no longer sold.
Which of the following types of income generally is subject to long-term capital-gains rates? A) Municipal bond coupon payments B) Corporate bond coupon payments C) Stock qualified dividends D) Savings account interest
C) Stock qualified dividends Generally, qualified dividends are subject to long-term capital-gains rates.
All the following statements regarding margin requirements for futures contracts are correct EXCEPT: A) Futures contracts generally have much lower margin requirements than the margin requirements of equities. B) The maintenance margin is always smaller than the initial margin. C) The maintenance margin is only required for sellers of futures contracts. D) The initial margin is the money placed with the clearinghouse when the trade is initially executed.
C) The maintenance margin is only required for sellers of futures contracts. The maintenance margin is required for both buyers and sellers of futures contracts.
A covered call is the most appropriate strategy for investors looking for: A) profit on substantial decreases in a stock's value. B) low purchase prices on a stock they anticipate will increase in value. C) additional income from a stock they already own. D) downside protection on a stock they already own.
C) additional income from a stock they already own. Selling covered calls on a stock an investor already owns can help the investor make additional income off the stock.
The board of directors announces the amount and date of the next dividend on the ____ date, while the ____ date is the first date on which the purchaser of a stock is no longer entitled to the recently declared dividend. A) declaration, record B) ex-dividend, record C) declaration, ex-dividend D) payment, record
C) declaration, ex-dividend The date of declaration is the announcement date, and the ex-dividend date is the date that new stock purchasers are not entitled to the dividend.
A mutual fund consisting mostly of large cap domestic stocks is most likely to have a: A) high beta relative to the S&P 500. B) low beta relative to the S&P 500. C) high R2 relative to the S&P 500. D) low R2 relative to the S&P 500.
C) high R2 relative to the S&P 500. A high R2 relative to the S&P 500 suggests that this portfolio is very similar to the S&P 500. Therefore, this portfolio is most likely to consist of large cap domestic stocks, the same securities that make up the S&P 500.
Morningstar will place a mutual fund into a value-style box if the mutual fund: A) has a low price per share. B) has a high net asset value (NAV). C) invests in firms with relatively low P/E ratios. D) invests in companies with relatively high market capitalizations.
C) invests in firms with relatively low P/E ratios. The value-style box refers to mutual funds that invest in firms that have relatively low P/E ratios, suggesting they provide a good value for their earnings.
Ingrid owns a bond that pays a semi-annual coupon. The last coupon was $20 and was paid 3 months ago. The current list price for the bond is $1,100. If Ingrid sells the bond, how much will she receive (ignoring any trading fees and taxes)? A) $1,080 B) $1,090 C) $1,100 D) $1,110
D) $1,110 Ingrid will receive the list price plus half of the accrued interest.
What is a convertible bond's conversion premium if its par value is $1,000, its current market price is $1,250, the company's current stock price is $40 a share, and the conversion ratio is 30? A) -$250 B) -$200 C) -$50 D) $50
D) $50 The conversion premium is $1,250 − (30 × $40) = $50.
An investor buys $50,000 worth of shares in a mutual fund. In the first year, the fund generates $1,000 of income. In the second year, it generates $2,000 of income. At the end of the 3rd year, the fund generates $500 of income and the shares are worth $55,000. What was this investor's holding period return for the past 3 years? A) 5.51% B) 6.36% C) 10.00% D) 17.00%
D) 17.00% HPR = [(Ending balance − Beginning balance) + income)]/Beginning balance HPR = [(55,000 − 50,000) + (1,000 + 2,000 + 500)]/50,000 HPR = [(5,000 + 3,500)/50,000 = 0.17, or 17%
Christie bought XYZ stock for $7 per share 3 months ago. She sold it today for $8 per share. What is her annualized return? A) 28% B) 31% C) 49% D) 71%
D) 71% This investor's return over the past three months was 14.29%, or ($8 − $7)/$7 = $1/$7 = 0.1429. Annualized return = (1 + 0.1429)4 − 1 = (1.1429)4 − 1 = 1.7060 − 1 = 0.7060, or about 71%
Which of the following is incorrect about the shape of distributions? A) Kurtosis is a measure of how heavy- or light-tailed data is relative to a normal distribution. B) A distribution that is more peaked than normal is leptokurtic. C) Skewness is a measure of the lack of symmetry for a data set. D) All the above are correct.
D) All the above are correct.
All the following statements regarding collateralized mortgage obligations (CMOs) are true EXCEPT: A) CMOs are subject to prepayment risk. B) CMOs allow investors to select the tranche that best meets their risk-return needs. C) CMOs are secured bonds. D) CMOs are issued by the U.S. government.
D) CMOs are issued by the U.S. government. CMOs are bonds created by private investment firms and are secured by a pool of mortgages. Since they are mortgage-backed securities, they are subject to prepayment risk. The advantage of a CMO is the ability to select the tranche that best meets the investor's risk-return needs.
Which of the following statements regarding mutual funds is (are) true? 1) In a typical year, mutual fund managers make substantial shifts to their funds' investments. 2) Mutual funds regularly issue new shares on the secondary market in anticipation of demand. A) I only B) II only C) Both I and II D) Neither I nor II
D) Neither I nor II Statement I is incorrect because mutual fund managers rarely make substantial shifts to their funds' investments. Statement II is incorrect because investors directly buy shares from the investment manager; shares are never available in the secondary market.
Which of the following statements about annual stock market returns for 1-year vs 20-year holding periods is (are) correct? 1) The mean annual return is much higher for 20-year than for 1-year holding periods. 2) The standard deviation of annual returns is much higher for 20-year than for 1-year holding periods. A) I only B) II only C) Both I and II D) Neither I nor II
D) Neither I nor II While the mean annual return for 20-year and 1-year holding periods are similar, the standard deviation of annual returns is much lower for 20-year than for 1-year holding periods.
All the following statements concerning the federal tax treatment of capital gains and losses are correct EXCEPT: A) Short-term capital gains are taxed at ordinary income rates. B) Long-term capital gains are taxed at multiple rates, depending on the taxpayer's marginal tax rate. C) Generally, up to $3,000 of net capital losses may be deducted from ordinary income with any remaining losses carried over to future tax years. D) Short-term capital losses are netted against unrealized capital gains in determining the taxable net gain.
D) Short-term capital losses are netted against unrealized capital gains in determining the taxable net gain. Short-term capital losses are netted against short-term and long-term capital gains when determining the taxable net gain. Unrealized capital gains are not taxed.
Which of the following positions has the greater risk for investors? A) Buying a call option B) Selling a covered call C) Selling a covered put D) Shorting a stock
D) Shorting a stock The loss for a long call is the premium paid. The risk for a covered call is to lose the stock, which is a loss of upside potential. A covered put involves writing a put for a stock that has been sold short. The put premium offsets some of the downside risk. The short is the riskiest position since the short seller must repurchase the stock in the open market if the stock price increases.
Donna is considering purchasing a 3-year bond that is selling for $1,000. Which of the following is correct if this bond has a 4% coupon, paid semiannually? A) The coupon rate is greater than the current yield. B) The current yield is greater than the YTM. C) The YTM is greater than the current yield. D) The YTM equals the coupon rate and current yield.
D) The YTM equals the coupon rate and current yield. A bond priced at par will have a YTM equal to the current yield and coupon rate.
Which of the following pieces of information is required to be included in a mutual fund's prospectus? A) The fund's future returns B) The fund's bid-ask spread C) The fund's investors D) The fund's past performance
D) The fund's past performance The Securities Exchange Commission requires mutual funds to include their past performance in their fund prospectus. Statement (A) is incorrect because future returns are volatile and unknown. Statement (B) is incorrect because the bid-ask spread is a transaction cost that, if applicable, is assessed by a third-party broker. Statement (C) is incorrect both to protect investors' privacy and because a fund's investors change so often that it would be impractical to include this information in the prospectus.
What is the primary risk to investors when hedging with futures contracts? A) The spot price may increase. B) Futures prices may increase. C) Futures prices may decrease. D) The investor has insufficient funds to meet margin requirements.
D) The investor has insufficient funds to meet margin requirements. The biggest risk when hedging with a futures contract is the required margin requirement funds necessary to maintain the hedged position. The purpose of the hedge is to mitigate the risk of price fluctuations.
All the following statements about all investment companies are correct EXCEPT: A) They provide investors with easier access to markets. B) They provide a diversified portfolio created more efficiently than if constructed from individual securities. C) They provide individual investors with investment opportunities otherwise available only to institutional investors. D) They provide a personal relationship with an investment manager.
D) They provide a personal relationship with an investment manager. Statement (D) is incorrect because investors will rarely have a personal relationship with the investment manager of an investment company; this is particularly true for investors in ETFs and closed-ended funds, which trade on the secondary market.
All the following are correct regarding common stock ownership EXCEPT: A) Most owners of common stock have the right to vote on corporate matters. B) With very few exceptions, shareholders are protected from personal liability from corporate obligations. C) Shareholder voting can be done in person or by proxy. D) When a corporation dissolves, the common stockholders are the first to be paid from the assets of the company.
D) When a corporation dissolves, the common stockholders are the first to be paid from the assets of the company. When a corporation liquidates, the secured creditors are the first to be paid, followed by unsecured creditors and preferred stockholders, and finally, common stockholders.
All the following are money market instruments EXCEPT: A) a banker's acceptance. B) a banker's acceptance. C) commercial paper. D) a Treasury bond.
D) a Treasury bond. Money market instruments are instruments with maturities of one year or less, which applies to all options except Treasury bonds that have much longer maturities.
Which of the following is a reason an investor may use derivative securities? A) to transfer their portfolio's risk to another investor. B) to hedge against price changes in another asset. C)to seek high returns on a speculative investment. D)all of the above.
D) all of the above. The primary purpose of derivatives is to transfer risk among market participants. Derivatives may be used to hedge against risk or to seek higher returns with a speculative investment.
An investor who sells a stock short: A) has limited risk. B) benefits when prices rise. C) profits when they identify undervalued securities. D) must cover any dividends payments made by the issuing company.
D) must cover any dividends payments made by the issuing company. Short positions have at least as much if not more risk than long positions in equity. Short positions benefit when prices fall, which is not likely to happen with undervalued stocks. When the issuing company pays a dividend, the short position must pay it.
The Western Fund generated a return of 10.4% over the last year. The relevant market portfolio had a return of 9.6% and the risk-free return was 2.1%. Western Fund's beta is 1.28. The Jensen's alpha for the fund is closest to: A) − 1.3. B) − 0.2. C) + 0.8. D) + 1.7.
α = rp - [rf + βp × (rm - rf)] α = 10.4 - [2.1 + 1.28 × (9.6 - 2.1)] = − 1.3